Difference between revisions of "2013 AMC 12B Problems/Problem 7"

(Created page with "==Problem== Jo and Blair take turns counting from 1 to one more than the last number said by the other person. Jo starts by saing "1", so Blair follows by saying "1, 2". Jo then ...")
 
(Solution 3)
 
(16 intermediate revisions by 12 users not shown)
Line 1: Line 1:
 +
{{duplicate|[[2013 AMC 12B Problems|2013 AMC 12B #7]] and [[2013 AMC 10B Problems|2013 AMC 10B #13]]}}
 +
 
==Problem==
 
==Problem==
Jo and Blair take turns counting from 1 to one more than the last number said by the other person. Jo starts by saing "1", so Blair follows by saying "1, 2". Jo then says "1, 2, 3", and so on. What is the <math>53^{rd}</math> number said?<br \>
+
Jo and Blair take turns counting from <math>1</math> to one more than the last number said by the other person. Jo starts by saying <math>``1"</math>, so Blair follows by saying <math>``1, 2"</math>. Jo then says <math>``1, 2, 3"</math>, and so on. What is the <math>53^{\text{rd}}</math> number said?<br \>
 +
 
 
<math>\textbf{(A)}\ 2 \qquad \textbf{(B)}\ 3 \qquad \textbf{(C)}\ 5 \qquad \textbf{(D)}\ 6 \qquad \textbf{(E)}\ 8</math>
 
<math>\textbf{(A)}\ 2 \qquad \textbf{(B)}\ 3 \qquad \textbf{(C)}\ 5 \qquad \textbf{(D)}\ 6 \qquad \textbf{(E)}\ 8</math>
 +
 +
==Solution==
 +
 +
We notice that the number of numbers said is incremented by one each time; that is, Jo says one number, then Blair says two numbers, then Jo says three numbers, etc.  Thus, after nine "turns", <math>1+2+3+4+5+6+7+8+9=45</math> numbers have been said.  In the tenth turn, the eighth number will be the 53rd number said, because <math>53-45=8</math>.  Since we are starting from 1 every turn, the 53rd number said will be <math>\boxed{\textbf{(E) }8}</math>.
 +
 +
==Faster Solution==
 +
 +
We notice that the number of numbers said is incremented by one each time; that is, Jo says one number, then Blair says two numbers, then Jo says three numbers, etc.  We notice that the number of numbers is <math>1 + 2 + 3 + 4 ...</math> every time we finish a "turn" we notice the sum of these would be the largest number <math>\frac{n(n+1)}{2}</math> under 53, we can easily see that if we double this it's <math>n^2 + n \simeq 106</math>, and we immediately note that 10 is too high, but 9 is perfect, meaning that at 9, 45 numbers have been said so far, <math>\frac{9(9+1)}{2} = 45</math> and <math>53 - 45 = \boxed{\textbf{(E) }8}</math>
 +
 +
==Solution 3==
 +
 +
Let <math>T(n)</math> denote the <math>n</math>th triangle number. Then, observe that the <math>T(n)</math>th number said is <math>n</math>. It follows that the <math>55</math>th number is <math>10</math> (as <math>55 = T(10)</math>). Thus, the <math>53</math>rd number is <math>10 - 2 = 8</math>, which is answer choice <math>\boxed{\textbf{(E)}}</math>.
 +
 +
~Mathavi
 +
 +
==Video Solution==
 +
https://youtu.be/a-3CAo4CoWc
 +
 +
~no one
 +
 +
== See also ==
 +
{{AMC10 box|year=2013|ab=B|num-b=12|num-a=14}}
 +
{{AMC12 box|year=2013|ab=B|num-b=6|num-a=8}}
 +
{{MAA Notice}}

Latest revision as of 16:18, 16 October 2022

The following problem is from both the 2013 AMC 12B #7 and 2013 AMC 10B #13, so both problems redirect to this page.

Problem

Jo and Blair take turns counting from $1$ to one more than the last number said by the other person. Jo starts by saying $``1"$, so Blair follows by saying $``1, 2"$. Jo then says $``1, 2, 3"$, and so on. What is the $53^{\text{rd}}$ number said?

$\textbf{(A)}\ 2 \qquad \textbf{(B)}\ 3 \qquad \textbf{(C)}\ 5 \qquad \textbf{(D)}\ 6 \qquad \textbf{(E)}\ 8$

Solution

We notice that the number of numbers said is incremented by one each time; that is, Jo says one number, then Blair says two numbers, then Jo says three numbers, etc. Thus, after nine "turns", $1+2+3+4+5+6+7+8+9=45$ numbers have been said. In the tenth turn, the eighth number will be the 53rd number said, because $53-45=8$. Since we are starting from 1 every turn, the 53rd number said will be $\boxed{\textbf{(E) }8}$.

Faster Solution

We notice that the number of numbers said is incremented by one each time; that is, Jo says one number, then Blair says two numbers, then Jo says three numbers, etc. We notice that the number of numbers is $1 + 2 + 3 + 4 ...$ every time we finish a "turn" we notice the sum of these would be the largest number $\frac{n(n+1)}{2}$ under 53, we can easily see that if we double this it's $n^2 + n \simeq 106$, and we immediately note that 10 is too high, but 9 is perfect, meaning that at 9, 45 numbers have been said so far, $\frac{9(9+1)}{2} = 45$ and $53 - 45 = \boxed{\textbf{(E) }8}$

Solution 3

Let $T(n)$ denote the $n$th triangle number. Then, observe that the $T(n)$th number said is $n$. It follows that the $55$th number is $10$ (as $55 = T(10)$). Thus, the $53$rd number is $10 - 2 = 8$, which is answer choice $\boxed{\textbf{(E)}}$.

~Mathavi

Video Solution

https://youtu.be/a-3CAo4CoWc

~no one

See also

2013 AMC 10B (ProblemsAnswer KeyResources)
Preceded by
Problem 12
Followed by
Problem 14
1 2 3 4 5 6 7 8 9 10 11 12 13 14 15 16 17 18 19 20 21 22 23 24 25
All AMC 10 Problems and Solutions
2013 AMC 12B (ProblemsAnswer KeyResources)
Preceded by
Problem 6
Followed by
Problem 8
1 2 3 4 5 6 7 8 9 10 11 12 13 14 15 16 17 18 19 20 21 22 23 24 25
All AMC 12 Problems and Solutions

The problems on this page are copyrighted by the Mathematical Association of America's American Mathematics Competitions. AMC logo.png